Last visit was: 23 Apr 2024, 15:03 It is currently 23 Apr 2024, 15:03

Close
GMAT Club Daily Prep
Thank you for using the timer - this advanced tool can estimate your performance and suggest more practice questions. We have subscribed you to Daily Prep Questions via email.

Customized
for You

we will pick new questions that match your level based on your Timer History

Track
Your Progress

every week, we’ll send you an estimated GMAT score based on your performance

Practice
Pays

we will pick new questions that match your level based on your Timer History
Not interested in getting valuable practice questions and articles delivered to your email? No problem, unsubscribe here.
Close
Request Expert Reply
Confirm Cancel
SORT BY:
Date
Tags:
Show Tags
Hide Tags
Math Expert
Joined: 02 Sep 2009
Posts: 92883
Own Kudos [?]: 618585 [11]
Given Kudos: 81563
Send PM
Most Helpful Reply
Math Expert
Joined: 02 Sep 2009
Posts: 92883
Own Kudos [?]: 618585 [1]
Given Kudos: 81563
Send PM
General Discussion
Director
Director
Joined: 09 Aug 2017
Posts: 689
Own Kudos [?]: 415 [0]
Given Kudos: 778
Send PM
Manager
Manager
Joined: 27 Dec 2016
Posts: 53
Own Kudos [?]: 61 [4]
Given Kudos: 59
Location: Bangladesh
GMAT 1: 610 Q43 V32
GRE 1: Q155 V145
GPA: 3.85
Send PM
Re: A careful review of hospital fatalities due to anesthesia during the [#permalink]
4
Kudos
Okay, so this is a flaw question. The hardest part about this question is not the stimulus, but rather, the abstract wording of the answer choices. When you come to a question with answer choices that have abstract wording, its best to move from wrong to right. Lets jump straight to the answer choices.

(B) is incorrect because the argument does not contain circular reasoning, or, as the answer choice states, the reasons given in support of the conclusion do not presuppose the truth of that conclusion. If you're having trouble understanding what "presupposing what the argument sets out to prove", means, click here for more information.
(C) is incorrect, and highly tempting (stupid abstract wording), because what its basically trying to say is that the evidence cited to show that a certain factor (not having equipment to monitor the patients O2 and Co2 levels) was absent when a certain result occurred (less people dying) does not show that the absence of that factor (not having the equipment) caused that result (less people dying).

In other words, not having the equipment does not show that less people dying occurred because of the absence of that equipment is what caused less people to die.

(Did you see how dumb that sounded?)

(D) is incorrect because the evidence is not inconsitent with any other information in the stimulus. The evidence (better training and equipment) is actually consistent. It would be easy to spot if it weren't, sort of like a paradox question.
(E) is incorrect because we don't have a third event so we don't even know what the heck this answer choice is talking about.

Lastly, (A) is the winner and your correct answer choice because the argument basically tries to say that having the equipment will not lead to significant cut in the fatalities simply because they already had a significant cut in fatalities when they had better training. You're probably asking yourself, "well wait, what if the new equipment, coupled with the better training leads to MORE lives being saved". And this, folks, is exactly what you should have asked yourself during this question. Answer choice (A) perfectly hits this point, in a more abstract sort of way.
Manager
Manager
Joined: 28 Mar 2018
Posts: 176
Own Kudos [?]: 73 [0]
Given Kudos: 63
Location: India
Concentration: Strategy, Technology
GMAT 1: 700 Q49 V36
GMAT 2: 730 Q50 V39
Send PM
Re: A careful review of hospital fatalities due to anesthesia during the [#permalink]
Hello experts generis abhi007 Bunuel , can you please help; why Option C is incorrect ?
It is very tempting choice and matches pre-thinking!
Manager
Manager
Joined: 21 Dec 2018
Posts: 66
Own Kudos [?]: 35 [0]
Given Kudos: 320
Location: India
GMAT 1: 630 Q45 V31
GMAT 2: 710 Q48 V40 (Online)
Send PM
Re: A careful review of hospital fatalities due to anesthesia during the [#permalink]
Jks3000 wrote:
Hello experts generis abhi007 Bunuel , can you please help; why Option C is incorrect ?
It is very tempting choice and matches pre-thinking!


I'll give it a try
The conclusion of the argument states that having the monitors would not help in the reduction of the fatalities. Nowhere does it mention that the absence caused the result i.e the reduction in fatalities. Thus C is wrong
User avatar
Non-Human User
Joined: 01 Oct 2013
Posts: 17206
Own Kudos [?]: 848 [0]
Given Kudos: 0
Send PM
Re: A careful review of hospital fatalities due to anesthesia during the [#permalink]
Hello from the GMAT Club VerbalBot!

Thanks to another GMAT Club member, I have just discovered this valuable topic, yet it had no discussion for over a year. I am now bumping it up - doing my job. I think you may find it valuable (esp those replies with Kudos).

Want to see all other topics I dig out? Follow me (click follow button on profile). You will receive a summary of all topics I bump in your profile area as well as via email.
GMAT Club Bot
Re: A careful review of hospital fatalities due to anesthesia during the [#permalink]
Moderators:
GMAT Club Verbal Expert
6917 posts
GMAT Club Verbal Expert
238 posts
CR Forum Moderator
832 posts

Powered by phpBB © phpBB Group | Emoji artwork provided by EmojiOne